Help me with this math problem

Help Me With This Math Problem

Answers

Answer 1

The required probabilities are as follows:

a) P(X=5) = 0.123

b) P(X6) = P(X=6+X=7+X=8+X=9+X=10)

c) P(X4)=P(X=0), P(X=1), P(X=2), and P(X=3)

Using a binomial random variable, the following is modeled:

There are always ten (10) persons that are being questioned.

From trial to trial, the likelihood of success (i.e., very little faith in newspapers) remains consistent (p=0.68).

The trials are impartial (a reasonable assumption, since the response of 1 of the people should not affect the response of any other)

There are just two possible results for each trial: low faith in newspapers or not.

X counts how many of the 10 trials were successful.

P(X=k) = is the probability mass function for the binomial random variable.

= [tex]^nC_k\times(p^k)\times(1-p)^{n-k}[/tex]

a) P(X=5) =[tex]^{10}C_5\times(0.68)^5\times(0.32)^5[/tex]≈ 0.123

b) P(X6) = P(X=6+X=7+X=8+X=9+X=10)

These may all be calculated individually, then summed together.

c) P(X4)=P(X=0), P(X=1), P(X=2), and P(X=3)

the same concept as in (b) above.

Learn more about probability learn-

brainly.com/question/11234923

#SPJ10


Related Questions

46. Machine A produces 500 springs a day. The number of defective springs produced by this machine each day is recorded for 60 days. Based on the distribution given below, what is the expected value of the number of defective springs produced by Machine A in any single day?

F. 0.00
G. 0.45
H. 0.70
J. 1.00
K. 1.50​

Answers

Answer:

G. 0.45

Step-by-step explanation:

To find expected value, you simply multiply the value of each outcome (the numbers in the left column) by its probability

(the numbers in the right column) and then add them all together.

0(0.7) + 1(0.2) + 2(0.05) + 3(0.05)

0 + 0.2 + 0.1 + 0.15 = 0.3 + 0.15 = 0.45

The expected value is 0.45. Thus, the correct option is G.

What is the expected value?

In parameter estimation, the expected value is an application of the weighted sum. Informally, the expected value is the simple average of a considerable number of individually determined outcomes of a randomly picked variable.

The expected value is given below.

E(x) = np

Where n is the number of samples and p is the probability.

The expected value is calculated as,

E(x) = 0 x 0.70 + 1 x 0.20 + 2 x 0.05 + 3 x 0.05

E(x) = 0 + 0.20 + 0.10 + 0.15

E(x) = 0.45

Thus, the correct option is G.

More about the expected value link is given below.

https://brainly.com/question/13945225

#SPJ2

Please solve this equation, I don't understand...

show all work!

Answers

Answer:

x = - 5/2

Step-by-step explanation:

Using the law of indices

. a^-n = 1/a^n

. (a^4)^5 = a^5×4 = a^20

Note : When two base are equal , we equate the Exponents.

3^4x-5 = (1/27)^2x+10

= 3^4x-5 = (27^-1)^2x+10

= 3^4x-5 = (3^-3)^2x+10

= 3^4x-5 = 3^ -3(2x+10)

= 3^4x-5 = 3^ -6x-30

The two base are 3 so we equate the exponents.

= 4x - 5 = -6x - 30

= 4x + 6x = -30+5

= 10x = -25

= 10x/10 = -25/10

x = -25/10

x = - 5/2

X is -5/2

Which number comes next in this series 1/64 1/32 1/16 1/8 1/4 1/2

Answers

Answer:

1/1

Step-by-step explanation:

As the question is halfing by 2

So

2 divide 2 equals 1

Answer:

1

Step-by-step explanation:

it's fractions of divided half. next one will be number 1

Which expression is equivalent to (a²b¹c)²(6a³b)(2c³)³₂ 4ab¹2c3 ?

Answers

The expression (a²b¹c)² × (6a³b) × (2c³)³ × (4ab¹) × (2c³) is equivalent to the expression 384a⁸b⁴c¹⁴.

What is an equivalent expression?

The equivalent is the expressions that are in different forms but are equal to the same value.

The expression is given below.

⇒ (a²b¹c)² × (6a³b) × (2c³)³ × (4ab¹) × (2c³)

By simplifying, we have

⇒ a⁴b²c² × 6a³b × 8c⁹ × 4ab¹ × 2c³

⇒ 384a⁸b⁴c¹⁴

More about the equivalent link is given below.

https://brainly.com/question/889935

#SPJ1

How do you solve 32-34?

Answers

You take 34 away From 32 and get -2

Answer:

32. {-4, 1, 2, 12}

33. {2, 6, 9, 31, 65}

34. No

Step-by-step explanation:

32. The domain of a relation is the set that contains all the x-coordinates of all the ordered pairs of the relation.

domain = {-4, 1, 2, 12}

33. The range of a relation is the set that contains all the y-coordinates of all the ordered pairs of the relation.

range = {2, 6, 9, 31, 65}

34. No since the same number, 2, appears twice as an x-coordinate. In a function, no two ordered pairs can have the same x-coordinate.

What is −7.6 − 2(2.2)

Answers

-12 hopefully this helps

Answer:

-12

Step-by-step explanation:

Order of operations:

PEMDAS

First, do parenthesis, 2 x 2.2 = 4.4 then do -7.6 - 4.4 = -12

Consider the proof.

Given: In △ABC, BD ⊥ AC
Prove: the formula for the law of cosines, a2 = b2 + c2 – 2bccos(A)

Triangle A B C is shown. A perpendicular bisector is drawn from point B to point D on side A C. The length of B C is a, the length of D C is b minus x, the length of A D is x, the length of A B is c, and the length of B D is h.




Statement

Reason
1. In △ABC, BD ⊥ AC 1. given
2. In △ADB, c2 = x2 + h2 2. Pythagorean thm.
3. In △BDC, a2 = (b – x)2 + h2 3. Pythagorean thm.
4. a2 = b2 – 2bx + x2 + h2 4. prop. of multiplication
5. a2 = b2 – 2bx + c2 5. substitution
6. In △ADB, cos(A) = StartFraction x Over c EndFraction 6. def. cosine
7. ccos(A) = x 7. mult. prop. of equality
8. a2 = b2 – 2bccos(A) + c2 8. ?
9. a2 = b2 + c2 – 2bccos(A) 9. commutative property
What is the missing reason in Step 8?

Pythagorean theorem
definition of cosine
substitution
properties of multiplication

Answers

The missing reason in Step 8 is substitution

When a triangle is not a right triangle and when either the lengths of two sides and the measurement of the included angle are known (SAS) or the lengths of the three sides are known (SSS), the Law of Cosines is used to discover the remaining pieces of the triangle.

According to the law of cosine, if a, b, and c are any triangle's three sides, then a² = b² + c² - 2bcosa

The x in statement 5 of the preceding proof is changed to c cos A from statement 6 in statement 8 of the proof.

Option C, from the list of alternatives, is the one that best explains statement 8.

Hence  missing reason in Step 8 is substitution

Learn more about substitution here:

https://brainly.com/question/190818

#SPJ10

Answer: OPTION C

Step-by-step explanation:

EDGE22

There were 582 students enrolled in a freshman-level chemistry class. By the end of the semester, the number of students who passed was 5 times the number of students who failed.

Answers

Answer:

let the no. of students who failed the examination be X

therefore, no. of students passed = 5x

total no. of students = 582

5x + X = 582

6x = 582

x = 97

no. of students failed the examination = 97

no. of students passed = 97*5 = 485

1
0/1 point
A student wants to write an expression for, "all of the elements which are not in set A but are in set B".

Answers

The way to write this expression in mathematics is A'∩B

How to solve for the expression

In order to get the right way to write this expression we have to break it down in two parts.

First we are told that some of the elements are not in A.

This is represented as A'.

Then we are told that they are in the set B. Hence we have it written as B.

Then the expression  not in set A but are in set B would be written as

A'∩B.

Read more on sets here:

https://brainly.com/question/13458417

#SPJ1

The Wilson family had 5 children. Assuming that the probability of a child being a girl is 0.5, find the probability that the Wilson family had

at least 3 girls?

at most 3 girls?

Answers

The expression represents the probability of getting exactly 3 heads is,

[tex]C(n,r)(0.5)^3(0.5)^6[/tex]

We have given

Aron flips a penny 9 times.

We have to determine

Which expression represents the probability of getting exactly 3 heads

What is binomial distribution?

The binomial distribution is determined as the probability of mass or discrete random variable which yields exactly some values.

The binomial probability formula shown has variables that represent:

[tex]=C\left(n,r\right)\times \left(p\right)^r\times\:\left(1-p\right)^{n-r}[/tex]

Where n is the total number of trials (here, we flip penny 9 times, hence n = 9).

r is the number we want to find (here, we want the probability of 3 heads, so r = 3).

p is the probability of success (here, success means getting heads.

So, in a coin flip the probability of heads is always 1/2, so p = 1/2).

Therefore, The expression represents the probability of getting exactly 3 heads is;

[tex]=C\left(n,r\right)\times \left(p\right)^r\times\:\left(1-p\right)^{n-r}[/tex]

[tex]=C\left(9,3\right)\times \left(0.5\right)^3\times \:\left(0.5\right)^6[/tex]

Hence, the expression represents the probability of getting exactly 3 heads is [tex]C\left(9,3\right)\times \left(0.5\right)^3\times \:\left(0.5\right)^6[/tex].

To learn more about Binomial Distribution visit:

brainly.com/question/7236644

#SPJ1

The probability that the Wilson family had at least 3 girls is 0.5, while the probability that Wilson family had at most 3 girls is 0.8125.

What is Binomial distribution?

A common discrete distribution is used in statistics, as opposed to a continuous distribution is called a Binomial distribution. It is given by the formula,

P(x) = ⁿCₓ (pˣ) (q⁽ⁿ⁻ˣ⁾)

Where,

x is the number of successes needed,

n is the number of trials or sample size,

p is the probability of a single success, and

q is the probability of a single failure.

Given the probability of a child being girl is 0.5, therefore, the probability of a child being boy is 0.5. Now, the probability of at least 3 girls out of 5 is,

Probability (x≥3)

= ⁵C₃ (0.5)³(0.5)² + ⁵C₄ (0.5)⁴(0.5)¹ + ⁵C₅(0.5)⁵(0.5)⁰

= 0.5

Probability (x≤3)

= ⁵C₃ (0.5)³(0.5)² + ⁵C₂(0.5)²(0.5)³ + ⁵C₁(0.5)¹(0.5)⁴ + ⁵C₀(0.5)⁰(0.5)⁵

= 0.8125

Hence, the probability that the Wilson family had at least 3 girls is 0.5, while the probability that Wilson family had at most 3 girls is 0.8125.

Learn more about Binomial Distribution:

https://brainly.com/question/14565246

#SPJ1

A farmer has 250 ft of fencing and wants to enclose a rectangular area of 3750 ft². What dimensions should she use?


The length of the longer side of the fence is:

The length of the shorter side of the fence is:

Answers

Answer:

Longer side is 75 feet, shorter side is 50 feet.

Step-by-step explanation:

Let l be the length of the longer side and w be the length of the shorter side.

2l + 2w = 250, which simplifies to

l + w = 125. Solving for w, we have

w = 125 - l.

lw = l(125-l) = 3750

[tex] {l}^{2} - 125l - 3750 = 0[/tex]

[tex]( l - 50)(l - 75) = 0[/tex]

l = 75, w = 50

Longer 75 , shorter 50

Simplify the expression -3z(1.8z-2.2).
-5.4z² + 6.6z
-5.4z²-6.6z
-1.2z²+5.2z
-1.2z²-5.2z
Mark this and return
Save and Exit
Next
Submit

Answers

Answer:

[tex] - 5.4z {}^{2} + 6.6z[/tex]

Step-by-step explanation:

Given:

[tex]-3z(1.8z-2.2)[/tex]

Solution:

Applying Distributive property,we obtain

[tex](−3z)(1.8z)+(−3z)(−2.2)[/tex]

Simplifying using PEMDAS:

[tex] - 5.4z {}^{2} + 6.6z[/tex]

Done!

Answer:A

Step-by-step explanation:

Determine the equation of the line that passes through the given points. (If you have a graphing calculator, you can use the table feature to confirm that the coordinates of both points satisfy your equation.) (-5, 10) and (5, -10) a. y = 2x-10 b. C. d. y = 2x y = -2x+ 20 y=-2x A​

Answers

Answer:

y = -2x

Step-by-step explanation:

It passes through -5, 10 and 5, -10

gradient = Δy/Δx

gradient = -10 - 10 / 5 -- 5

gradient = -20 / 10

gradient = -2

let's use -5, 10 for y = mx + b

10 = -2(-5) + b

10 = 10 + b

b = 10 - 10

b = 0.

the equation of the line is just y = -2x

PLEASE HELP AND SHOW WORK PLEASE.

How is this a no solution answer? I came out with the answer x < -2 and x ≥ 5


5 - x > 7 and 2x + 3 ≥ 13

Answers

Inequalities help us to compare two unequal expressions. There exists no solution to the given set of inequalities.

What are inequalities?

Inequalities help us to compare two unequal expressions. Also, it helps us to compare the non-equal expressions so that an equation can be formed. It is mostly denoted by the symbol <, >, ≤, and ≥.


The given Inequalities can be solved as,

5 - x > 7

-x > 7 - 5

-x > 2

x < -2

2x + 3 ≥ 13

2x ≥ 10

x ≥ 5

As per the solution of the two inequalities, the value of x should be less than -2 but at the same time, it should be more than or equal to 5, which is impossible. Thus, there is no solution for the given inequalities.

This can be confirmed by graphing the two inequalities, as shown below. Since there is no area in common between the two inequalities, there exists no solution to the given set of inequalities.

Learn more about Inequality:

https://brainly.com/question/19491153

#SPJ1

Suppose that
f
(
x
,
y
)
=
x
+
5
y
f
(
x
,
y
)
=
x
+
5
y
at which

1

x

1
,

1

y

1
-
1

x

1
,
-
1

y

1
.

Absolute minimum of
f
(
x
,
y
)
f
(
x
,
y
)
is
Absolute maximum of
f
(
x
,
y
)
f
(
x
,
y
)
is

Answers

Answers:Absolute min = -6Absolute max = 6

========================================================

Explanation:

The range of x values is [tex]-1 \le x \le 1[/tex] which means x = -1 is the smallest and x = 1 is the largest possible.

Similarly the smallest y value is y = -1 and the largest is y = 1.

----------

Plug in the smallest x and y value to get

f(x,y) = x+5y

f(-1,-1) = -1+5(-1)

f(-1,-1) = -6

Therefore, the absolute min is -6

----------

Now plug in the largest x and y values

f(x,y) = x+5y

f(1,1) = 1+5(1)

f(1,1) = 6

The absolute max is 6

WILL GIVE THE BRAINLIEST!

If r and s are real numbers such that
r>s>0, then

(A) -s>-r> 0.
(B) 0>-r> -s.
(C) 0> -s> -r.
(D) -s>0> -r.
(E) -r> -s > 0.

Answers

Answer:  Choice (C)  0> -s> -r

==========================================================

Explanation:

Let's consider an example. Pick any two positive numbers for r and s, where r is larger than s. I'll go with these:

r = 5s = 2

We see that r > s is true since 5 > 2 is true

Also both are larger than 0 so we can say r > 0 and s > 0

This all combines into r > s > 0 being the case.

If we negate each number, then -r > -s would no longer be true. Use a number line to see that -5 is to the left of -2, so -5 > -2 is not true. Instead it would be -5 < -2

You can think of it like floors on a skyscraper. Positive numbers are above ground, while negative numbers are below ground in the basement levels. See the diagram below.

So -r > -s must be -r < -s instead. Both are smaller than 0

-r < 0 and -s < 0

This combines to -r < -s < 0

Flip everything and swap the outer sides to get 0 > -s > -r which leads us to choice C as the final answer

Please help and fast for brainliest

The graph shows the number of gallons of white paint that were mixed with gallons of green paint in various different ratios:

Draw the graph on a grid. The title is Mixing Paint. The horizontal axis label is Green Paint in gallons. The scale is 0 to 24 in increments of 3. The vertical axis label is White Paint in gallons. The scale is 0 to 72 in increments of 9. Plot points at the ordered pairs 3,9 and 6,18 and 9,27.

The number of gallons of white paint mixed with 1 gallon of green paint is ______.

Answers

Answer:

3 gallons

Step-by-step explanation:

For every value of green ( 'x') ....white is 3 times as much

  for 1 gallon green     ====> 3 gallons white

Find the slope of the line passing through the points (3,-9) and (2, -3).

Answers

Answer:

The slope of the line passing through is -6.

Prove triangle ABC is congruent to triangle DEC.

Answers

Triangle ABC and DEC are said to congruent since all their sides and angles are equal.

How to prove the statement

The angle for triangle ABC lies in angle B

The angle for triangle DEC lies in angle E

From the diagram, angles B and E are alternate angles and alternate angles are equal.

A corresponds to D

B corresponds to E

The measure of their angles are also equal

Note, congruent triangles are triangles with three corresponding sides and angles

Therefore, triangle ABC and DEC are said to congruent since all their sides and angles are equal.

Learn more about congruent triangles here:

https://brainly.com/question/1675117

#SPJ1

A car has a maximum speed of 355.7 feet/second. Convert this speed to miles/hour.

Answers

Answer:

242.5226626 mph

Step-by-step explanation:

1 ft/s = 0.681818 mph

355.7 * 0.681818 = 242.5226626

355.7 ft/s = 242.5226626 mph

what of the following points are on the line given by the equation y = 5x

Answers

(0,0) and (1,5). The slope is 5/1. So put it to any point and go up 5 and right 1 or down 5 and left 1.

If f(x) = x + 8 and g(x) = –4x – 3, find (f + g)(x).
A. (f + g)(x) = 5x + 11
B. (f + g)(x) = –3x + 5
C. (f + g)(x) = –5x – 11
D. (f + g)(x) = 3x – 5

Answers

Answer:

B

Step-by-step explanation:

(f + g)(x)

= f(x) + g(x)

= x + 8 - 4x - 3 ← collect like terms

= - 3x + 5

Answer:

[tex]\huge\boxed{\sf (f + g)(x) = -3x + 5}[/tex]

Step-by-step explanation:

Given functions:f(x) = x + 8g(x) = -4x - 3Solution:

Add both functions

(f + g)(x) = x + 8 + (-4x - 3)

(f + g)(x) = x + 8 - 4x - 3

(f + g)(x) = x - 4x + 8 - 3

(f + g)(x) = -3x + 5

[tex]\rule[225]{225}{2}[/tex]

If 3x − 6 ≤ f(x) ≤ x2 − 3x + 3 for x ≥ 0, find lim x→3 f(x).

Answers

Answer:

Step-by-step explanation:

Given mCT=26 and m/CAT =124° find the length of CA, the radius in Circle A. Use
π = 3.14 in your calculation and round to the nearest tenth.

Answers

Answer:

[tex]\text{length of \textit{CA}} \ = \ 12.0 \ \ \ (\text{nearest tenth})[/tex]

Step-by-step explanation:

Radian measure is the ratio of the length of a circular arc to its radius.

A radian is the measurement of the central angle which subtends an arc whose length is equal to the length of the radius of the circle.

In the case of the unit circle, as shown in the figure below, one radian is the angle of the sector with a radius of 1 and circular arclength of 1.

Following this definition, the magnitude, in radians, of one complete revolution of a unit circle is the circumference of the unit circle divided by its radius, [tex]\displaystyle\frac{2\pi}{1}[/tex] or [tex]2\pi[/tex]. Thus, [tex]2\pi[/tex] radians is equal to [tex]360^{\circ}[/tex] degrees. Alternatively, one radian is equal to [tex]\displaystyle\frac{180^{\circ}}{\pi} \ \approx \ 57.296^{\circ} \ \ \ (3 \ d.p.)[/tex].

Since radian measure is defined as the ratio of the arc length of a sector to its radius, hence

                                                        [tex]\displaystyle\frac{s}{r} \ = \ \theta \\\\ s \ = \ r\theta[/tex]

where [tex]s[/tex] is the arclength, [tex]r[/tex] is the radius, and [tex]\theta[/tex] is the central angle, in radians.

Therefore, the length of CA is

                                           [tex]\displaystyle\frac{s}{\theta} \ = \ r \\ \\ r \ = \ \displaystyle\frac{26}{124^{\circ} \ \times \ \displaystyle\frac{\pi}{180^{\circ}} \ \text{rad}} \\ \\ r = \displaystyle\frac{26 \ \times \ 45}{31 \ \times \ 3.14} \\ \\ r\ = \ 12.0 \ \ \ \ \left(\text{nearest tenth}\right)[/tex]

Which expression can be used to find the difference of the polynomials?

Answers

The expression that can be used to find the difference of the polynomials is (4m - 5) + ( -6m + 7 - 2n). D

How to find the difference

Given the expression:

(4m - 5) - (6m - 7 + 2n)

Note that  - * - = +

+ * - = -

So in finding the difference, we have

(4m - 5) - ( 6m + 7 - 2n )

It could as be written as

(4m - 5) + ( -6m + 7 - 2n)

Therefore, the expression that can be used to find the difference of the polynomials is (4m - 5) + ( -6m + 7 - 2n) . D

Learn more about polynomials here:

https://brainly.com/question/4142886

#SPJ1

Think about all of the ways in which a circle and a parabola can intersect.
Select all of the number of ways in which a circle and a parabola can intersect.
00
1
2
03
04
05
DONE

Answers

There will be four ways a circle and a parabola can intersect because the solution of the quartic equation will be 4.

What is a circle?

It is described as a set of points, where each point is at the same distance from a fixed point (called the center of a circle)

We have a circle and a parabola:

As we know the standard form of a circle:

[tex]\rm (x-h)^2 +(y-k)^2=r^2[/tex]

The standard form of the parabola:

[tex]\rm y = a(x-h)^2+k[/tex]  

If we plug the value of y from the parabola equation in the circle equation, we get a quartic equation(4th order equation)

The solution of the quartic equation will be 4.

Thus, there will be four ways a circle and a parabola can intersect because the solution of the quartic equation will be 4.

Learn more about circle here:

brainly.com/question/11833983

#SPJ1

Select all the correct graphs.
Choose the graphs that indicate equations with no solution.

Answers

Answer:

First Graph:  -2x - 1 = 3(^-x)

Last Graph: 2^(-x) + 2 = 5^-x + 3

Step-by-step explanation:

For a system of equations to have a solution set, the graphs that depict them must intersect at one point.

Both graphs #1 and #5 do not intersect, hence graphs #1 and #5 are the only graphs that do not have solutions while the other graphs do.

1) When the polynomial P(x) is divided by x+5, the remainder is 3. Which of the following is definitely true?
a) P(3) = 5
b) P(-5) = 3
c) P(5) = 3
d) P(-3) = 5

2) P is a degree 3 polynomial with real coefficients and three zeros. Two of the zeros are 5 and 3+4i. What is the other zero?
a) 4-3i
b) 4+3i
c) -5
d) 3-4i

Answers

1) The definitely true for the statement is b) P(-5) = 3.

2) The three roots are 5, 3 + 4i, and 3 - 4i. Option D is correct.

What is the remainder theorem for polynomials?

If there is a polynomial p(x), and a constant number 'a', then

[tex]\dfrac{p(x)}{(x-a)} = g(x) + p(a)[/tex]

where g(x) is a factor of p(x)

Given;

1) When the polynomial P(x) is divided by x+5, the remainder is 3.

If a polynomial P(x) is divided by (x-a), then the remainder is P(a).

If the polynomial P(x) is divided by (x+5), then the remainder will be P(-5).

So, P(-5) = 3

Therefore, the definitely true for the statement is b) P(-5) = 3.

2) P is a 3 degree polynomial with real coefficients and three zeros. Two of the zeros are 5 and 3+4i.

The complex roots always exist in conjugate pairs so,

3 + 4i and 3 - 4i

Thus, the three roots are 5, 3 + 4i, and 3 - 4i. Option D is correct.

Learn more about polynomial :

https://brainly.com/question/16399195

#SPJ1

There is a mound of g pounds of gravel in a quarry. Throughout the day, 200 pounds of gravel is added to the mound. Two orders of 700 pounds are sold and the gravel is removed from the mound. At the end of the day, the mound has 1,200 pounds of gravel.

Answers

Answer:

[tex]\fbox {2,400 pounds}[/tex]

Step-by-step explanation:

Information we have :

200 pounds is added2 orders of 700 pounds is removed1,200 pounds remain

What we need to find :

Original amount

Solving :

g + 200 - 2(700) = 1200g + 200 = 1200 + 1400g + 200 = 2600g = 2,400 pounds of gravel

For the first 8 months at his new job, James gets
an increase in pay. What is the rate at which
James receives a pay increase? Approximately
what was his starting pay? Write an equation to
model the relationship.

Answers

James receives a pay increase of 100% per month and has a starting salary of $500.

What is an equation?

An equation is an expression that shows the relationship between two or more numbers and variables.

Let b represent the rate of pay increase and a represent the initial pay.

Let us assume that he had received 128000 after the 8 months. hence:

128000 = ab⁸   (1)

Also in the 10th month, he had received 512000, hence:

512000 = ab¹⁰    (2)

From the both equations:

a = 500, b = 2

rate of increase = 200% - 100% = 100%

James receives a pay increase of 100% per month and has a starting salary of $500.

Find out more on equation at: https://brainly.com/question/2972832

#SPJ1

Other Questions
Tech A says that on the full floating axle, there are two tapered roller bearings between the hub and the outside of the axle housing. Tech B says that a semi-floating axle, the axle supports the weight of the vehicle and is also subject to bending forces as the vehicle corners. Who is correct Put the following events of the Russian Revolution in chronological order.Lenin and the Bolsheviks stage a coup in theOctober Revolution.Vladimir Lenin returns from exile.Protesters march in Petrograd during theFebruary Revolution.Czar Nicholas II abdicates the throne.Russia signs a peace treaty with Germany. Joselyn spends all of her free time on the Internet. She fears missing updates from her favorite celebrities and texts from her friends. She used to take a kickboxing class after school, but she quit the class because she feels like she can't be away from her phone for the time it takes for the class and after-class shower. Joselyn's use of the Internet parallels symptoms found in: tag questions she has a laptop is this sentence correct?"he became unclear when he murmured something that she could not quite catch" Why is it important to discuss promptly the results of a supplier visit or survey with the supplier? If a supplier has a weak area, under what conditions would supplier development be appropriate? Which of the objects around you is following Newton's first law of motion? What is the typical relationship between satisfaction and loyalty? On a coordinate plane, a right triangle has points A (negative 5, negative 6), B (negative 5, negative 2), C (negative 2, negative 2). Examine the right triangle ABC. Which rise and run would create a similar right triangle on the same line? a rise of 6 and a run of 8 a rise of 8 and a run of 6 a rise of 6 and a run of 5 a rise of 5 and a run of 6 When an application has multiple uses what is it known as?. What are some examples of security goals that you may have for an organization? Which particles that make up an atom are involved in nuclear reactions?. The French were convinced to support the American war against the British after the Americans won the Battle ofTrenton.Saratoga.Concord.Yorktown. write a difference between shikism and kirant dharma . La verdad es importante. Voy a la decir ahora. Ted is trying to change 0.43 to a fraction. Evaluate teds method so far The position of given object is s(t)= t^5 + cos 4tFind the velocity and acceleration functions both of the drop menus have the same options 5The recursive formula for a geometric sequence is given below. What is the third term in the sequence?f(1) = 2f(n) = 3f(n 1)-OA. 18OB. 54OC. 162OD. 12ResetSubmit Click to review the online content. Then answer the question(s) below, using complete sentences. Scroll down to view additional questions.Online Content: Site 1What factors should be considered when putting together a personal fitness program?